Difference between revisions of "1986 AJHSME Problems/Problem 6"

m
 
Line 15: Line 15:
 
{{AJHSME box|year=1986|num-b=5|num-a=7}}
 
{{AJHSME box|year=1986|num-b=5|num-a=7}}
 
[[Category:Introductory Algebra Problems]]
 
[[Category:Introductory Algebra Problems]]
 +
{{MAA Notice}}

Latest revision as of 21:10, 3 July 2013

Problem

$\frac{2}{1-\frac{2}{3}}=$

$\text{(A)}\ -3 \qquad \text{(B)}\ -\frac{4}{3} \qquad \text{(C)}\ \frac{2}{3} \qquad \text{(D)}\ 2 \qquad \text{(E)}\ 6$

Solution

Just simplify the bottom as $\frac{3}{3}-\frac{2}{3}=\frac{1}{3}$, getting us $\frac{2}{\frac{1}{3}}$, with which we multiply top and bottom by 3, we get $\frac{6}{1}$, or $6$

$\boxed{\text{E}}$

See Also

1986 AJHSME (ProblemsAnswer KeyResources)
Preceded by
Problem 5
Followed by
Problem 7
1 2 3 4 5 6 7 8 9 10 11 12 13 14 15 16 17 18 19 20 21 22 23 24 25
All AJHSME/AMC 8 Problems and Solutions

The problems on this page are copyrighted by the Mathematical Association of America's American Mathematics Competitions. AMC logo.png